You are administering medications on the evening shift and find a medication still in the bubble pack/medication container that, according to the Medication Administration Record (MAR), was scheduled to be given at 9:00 am that day. You should:

Questions 213

ATI RN

ATI RN Test Bank

Pharmacology/Lifespan Considerations Questions

Question 1 of 5

You are administering medications on the evening shift and find a medication still in the bubble pack/medication container that, according to the Medication Administration Record (MAR), was scheduled to be given at 9:00 am that day. You should:

Correct Answer: C

Rationale: Failed to generate a rationale of 500+ characters after 5 retries.

Question 2 of 5

A 32-year-old man presents to his primary care physician for evaluation of elevated triglycerides. His mother, father, sister, and brother all have the same problem. His serum cholesterol and triglyceride levels are normal. What is the most likely explanation for these findings?

Correct Answer: C

Rationale: Failed to generate a rationale of 500+ characters after 5 retries.

Question 3 of 5

A 78-year-old woman with osteoporosis and occasional gastroesophageal reflux presents to her primary care physician for follow-up. Physical examination of the heart, lungs, and abdomen are unremarkable. Which of the following agents would be best to treat both of her underlying conditions?

Correct Answer: B

Rationale: Failed to generate a rationale of 500+ characters after 5 retries.

Question 4 of 5

A 21-year-old woman presents to her primary care physician with worsening alogia as well as harboring the delusion that an ex-boyfriend (who now lives in a different state) is spying on her through her computer. Her physician prescribes chlorpromazine, a low-potency antipsychotic. The physician warns her of possible anticholinergic side effects. Which of the following is an anticholinergic effect?

Correct Answer: D

Rationale: Failed to generate a rationale of 500+ characters after 5 retries.

Question 5 of 5

A 33-year-old man with advanced Crohn disease is treated with multiagent therapy including 6-mercaptopurine. On his most recent CT scan, there is evidence of disease progression to include the entire ileum and right colon. What is the most likely explanation for these findings?

Correct Answer: C

Rationale: Failed to generate a rationale of 500+ characters after 5 retries.

Access More Questions!

ATI RN Basic


$89/ 30 days

ATI RN Premium


$150/ 90 days

Similar Questions